LSAT and Law School Admissions Forum

Get expert LSAT preparation and law school admissions advice from PowerScore Test Preparation.

User avatar
 Dave Killoran
PowerScore Staff
  • PowerScore Staff
  • Posts: 5852
  • Joined: Mar 25, 2011
|
#89208
Complete Question Explanation
(The complete setup for this game can be found here: lsat/viewtopic.php?f=252&p=89207)

The correct answer choice is (D)

This is a classic final game question—time-consuming and frustrating. If S is performed fifth, then either O or T must apparently be performed fourth. But, T cannot be performed fourth because there is no workable hypothetical with T fourth and S fifth (N must then be performed second, V must be performed first, and then O is left to be performed third, creating a V-N-O-T-S hypothetical that violates the first rule). Hence, O must be performed fourth, and this inference eliminates answer choices (B) and (C). Now that we have established that O will be performed fourth and S will be performed fifth, we can examine the templates to see if any possibilities have been eliminated. Because Template #4 features S as the first performance, we can eliminate Template #4 from consideration. The remaining three templates each feature N as either the first or second performance, and thus we can eliminate answer choice (A). The only remaining answer choices are (D) and (E)—V performed either first or second. Of our three templates (#1, #2, and #3), only Template #2 features V as either the first or second performance, and thus Template #2—which features V as the first performance—proves answer choice (D) correct.

Get the most out of your LSAT Prep Plus subscription.

Analyze and track your performance with our Testing and Analytics Package.